FINANCE final exam ch 10, 11, 12, & 13 test bank

¡Supera tus tareas y exámenes ahora con Quizwiz!

The expected return on a security depends on which of the following? I. risk-free rate of return II. amount of the security's unique risk III market rate of return IV. standard deviation of returns A. I and III only B. II and IV only C. II, III, and IV only D. I, III, and IV only E. I, II, III, and IV

A. I and III only

Which of the following will increase the value of a levered firm according to M&M Proposition I, with taxes? I. decrease in the amount of the debt II. increase in the value of the unlevered firm III. decrease in the tax rate IV. increase in the interest rate on the debt A. II only B. I and IV only C. II and III only D. II and IV only E. II, III, and IV only

A. II only

Which one of the following had the lowest standard deviation of returns for the period of 1926 - 2008? A. U.S. Treasury bill B. Inflation C. Long-term corporate bonds D. Large-company stocks E. Long-term government bonds

A. U.S. treasury bill

Which one of the following statements is correct? Assume the pre-tax cost of debt is less than the cost of equity. A. A firm may change its capital structure if the government changes its tax policies. B. A decrease in the dividend growth rate increases the cost of equity. C. A decrease in the systematic risk of a firm will increase the firm's cost of capital. D. A decrease in a firm's debt-equity ratio will decrease the firm's cost of capital. E. The cost of preferred stock decreases when the tax rate increases.

A. a firm may change its capital structure if the government changes its tax policies

Assume both corporate taxes and financial distress costs apply to a firm. Given this, the static theory of capital structure illustrates that: A. a firm's value and its weighted average cost of capital are inversely related. B. a firm's value and its tax rate are inversely related. C. the maximum value of a firm is obtained when a firm is financed solely with debt. D. the value of a firm rises as the interest rate on debt rises. E. the value of a firm rises as both the interest rate on debt and the tax rate rise.

A. a firm's value and its weighted average cost of capital are inversely related

The variance is the average squared difference between which of the following? A. Actual return and average return B. Actual return and (average return/N - 1) C. Actual return and the real return D. Average return and the standard deviation E. Actual return and the risk-free rate

A. actual return and average return

32. A firm uses its weighted average cost of capital to evaluate the proposed projects for all of its varying divisions. By doing so, the firm: A. automatically gives preferential treatment in the allocation of funds to its riskiest division. B. encourages the division managers to only recommend their most conservative projects. C. maintains the current risk level and capital structure of the firm. D. automatically maximizes the total value created for its shareholders. E. allocates capital funds evenly amongst its divisions.

A. automatically gives preferential treatment in the allocation of funds to its riskiest division

Mary owns a risky stock and anticipates earning 16.5 percent on her investment in that stock. Which one of the following best describes the 16.5 percent rate? A. Expected return B. Real return C. Market rate D. Systematic return E. Risk premium

A. expected return

Which one of the following terms is inclusive of both direct and indirect bankruptcy costs? A. Financial distress costs B. Capital structure costs C. Financial leverage D. Homemade leverage E. Cost of capital

A. financial distress costs

Which one of the following is defined as the average compound return earned per year over a multiyear period? A. Geometric average return B. Variance of returns C. Standard deviation of returns D. Arithmetic average return E. Normal distribution of returns

A. geometric average return

Which one of the following will increase the cost of equity, all else held constant? A. Increase in the dividend growth rate B. Decrease in beta C. Decrease in future dividends D. Increase in stock price E. Decrease in market risk premium

A. increase in the dividend growth rate

Which one of the following terms applies to the costs incurred by a firm which is trying to avoid filing for bankruptcy? A. Indirect bankruptcy costs B. Direct bankruptcy costs C. Static theory cost D. Optimal capital structure cost E. Reorganization costs

A. indirect bankruptcy costs

Over the period of 1926-2008: A. long-term government bonds underperformed long-term corporate bonds. B. small-company stocks underperformed large-company stocks. C. inflation exceeded the rate of return on U.S. Treasury bills. D. U.S. Treasury bills outperformed long-term government bonds. E. large-company stocks outperformed all other investment categories.

A. long-term government bonds underperformed long-term corporate bonds

The lower the standard deviation of returns on a security, the _____ the expected rate of return and the _____ the risk. A. lower; lower B. lower; higher C. higher; lower D. higher; higher E. You cannot determine anything about the expected rate of return from the standard deviation.

A. lower; lower

If a security plots to the right and below the security market line, then the security has ____ systematic risk than the market and is _____. A. more; overpriced B. more; underpriced C. less; overpriced D. less; underpriced E. less; correctly priced

A. more; overpriced

Which one of the following describes systemic risk? A. Risk that affects a large number of assets B. An individual security's total risk C. Diversifiable risk D. Asset specific risk E. Risk unique to a firm's management

A. risk that affects a large number of assets

Which one of the following categories has the widest frequency distribution of returns for the period 1926-2008? A. Small-company stocks B. U.S. Treasury bills C. Long-term government bonds D. Inflation E. Large-company stock

A. small-company stocks

Which one of the following is the positive square root of the variance? A. Standard deviation B. Mean C. Risk-free rate D. Average return E. Real return

A. standard deviation

Which one of the following statements is correct concerning both the dollar return and the percentage return on a stock investment? A. The dollar return is dependent on the size of the investment while the percentage return is not. B The dollar return is more accurate than the percentage return because the dollar return includes . dividend income while the percentage return does not. C. The dollar return considers the time value of money while the percentage return does not. D. Dollar returns are based on capital gains while percentage returns are based on the total rate of return. E. Dollar returns must either be zero or a positive value while percentage returns can be negative, zero, or positive.

A. the dollar return is dependent on the size of the investment while the percentage return is not

All else constant, the weighted average cost of capital for a risky, levered firm will decrease if: A. the firm's bonds start selling at a premium rather than at a discount. B. the market risk premium increases. C. the firm replaces some of its debt with preferred stock. D. corporate taxes are eliminated. E. the dividend yield on the common stock increases.

A. the firm's bonds start selling at a premium rather than at a discount

A risky security has less risk than the overall market. What must the beta of this security be? A. 0 B. > 0 but <1 C. 1 D. > 1 E. The beta cannot be determined based on the information provided.

B. > 0 but <1

Boone Brothers remodels homes and replaces windows. Ace Builders constructs new homes. If Boone Brothers considers expanding into new home construction, it should evaluate the expansion project using which one of the following as the required return for the project? A. Boone Brothers' cost of capital B. Ace Builders' cost of capital C. Average of Boone Brothers' and Ace Builders' cost of capital D. Lower of Boone Brothers' or Ace Builders' cost of capital E. Higher of Boone Brothers' or Ace Builders' cost of capital

B. Ace Builders' cost of capital

The aftertax cost of which of the following are affected by a change in a firm's tax rate? I. preferred stock II. debt III. equity IV. capital A. I and III only B. II and IV only C. I, II, and IV only D. II, III, and IV only E. I, II, III, and IV

B. II and IV only

Which of the following statements correctly relate to M&M Proposition I, with taxes? I. Debt decreases the value of a firm. II. The levered value of a firm exceeds the firm's unlevered value. III. The weighted average cost of capital (WACC) is constant. IV. The optimal capital structure is zero debt. A. I only B. II only C. II and III only D. I and IV only E. I, III, and IV only

B. II only

Which of the following will increase the cost of equity for a firm with a beta of 1.1? I. decrease in the security's beta II. decrease in the market risk premium III. decrease in the risk-free rate IV. increase in the risk-free rate A. II only B. III only C. I and II only D. II and III only E. I and IV only

B. III only

Which one of the following is minimized when the value of a firm is maximized? A. Return on equity B. WACC C. Debt D. Taxes E. Bankruptcy costs

B. WACC

27. Which one of the following statements is correct, all else held constant? A. Beta is used to compute the return on equity and the standard deviation is used to compute the return on preferred. B. A decrease in a firm's WACC will increase the attractiveness of the firm's investment options. C. The aftertax cost of debt increases when the market price of a bond increases. D. If you have both the dividend growth and the security market line's costs of equity, you should use the . higher of the two estimates when computing WACC. E. WACC is only applicable to firms that issue both common and preferred stock.

B. a decrease in a firm's WACC will increase the attractiveness of the firm's investment options

Which one of the following statements related to the security market line is correct? A. An underpriced security will plot below the security market line. B. A security with a beta of 1.54 will plot on the security market line if it is correctly priced. C. A portfolio with a beta of 0.93 will plot to the right of the overall market. D. A security with a beta of 0.99 will plot above the security market line if it is correctly priced. E. A risk-free security will plot at the origin.

B. a security with a beta of 1.54 will plot on the security market line if it is correctly priced

Which one of the following is the best example of unsystematic risk? A. Inflation exceeding market expectations B. A warehouse fire C. Decrease in corporate tax rates D. Decrease in the value of the dollar E. Increase in consumer spending

B. a warehouse fire

In the process of liquidation, some types of claims receive preference over other claims. Which one of the following determines which type of claim is paid first? A. Technical insolvency definition B. Absolute priority rule C. Accounting insolvency definition D. Chapter 7 of the Federal Bankruptcy Reform Act of 1978 E. Securities and Exchange Commission

B. absolute priority rule

Old Town Industries has three divisions. Division X has been in existence the longest and has the most stable sales. Division Y has been in existence for five years and is slightly less risky than the overall firm. Division Z is the research and development side of the business. When allocating funds, the firm should probably: A. require the highest rate of return from division X since it has been in existence the longest. B. assign the highest cost of capital to division Z because it is most likely the riskiest of the three divisions. C. use the firm's WACC as the cost of capital for division Z as it provides analysis for the entire firm. D use the firm's WACC as the cost of capital for divisions A and B because they are part of the revenue- . producing operations of the firm. E. allocate capital funds evenly amongst the divisions to maintain the current capital structure of the firm.

B. assign the highest cost of capital to division Z because it is most likely the riskiest of the three divisions

Which one of the following statements is correct? A. A prepack is a plan of liquidation used to distribute a firm's assets. B. Bankruptcy courts have "cram-down" powers. C. The absolute priority rule must be strictly followed in all bankruptcy proceedings. D. Creditors cannot force a firm into bankruptcy even though they might like to do so. E. A reorganization plan can only be approved if the firm's creditors all agree with the plan.

B. bankruptcy courts have "cram-down" powers

Which one of the following measures the amount of systematic risk present in a particular risky asset relative to that in an average risky asset? A. Squared deviation B. Beta coefficient C. Standard deviation D. Mean E. Variance

B. beta coefficient

2. Lester lent money to The Corner Store by purchasing bonds issued by the store. The rate of return that he and the other lenders require is referred to as the: A. pure play cost. B. cost of debt. C. weighted average cost of capital. D. subjective cost. E. cost of equity.

B. cost of debt

3. The weighted average cost of capital is defined as the weighted average of a firm's: A. return on its investments. B. cost of equity and its aftertax cost of debt. C. pretax cost of debt and equity securities. D. bond coupon rates. E. dividend and capital gains yields.

B. cost of equity and its aftertax cost of debt

The level of financial risk to which a firm is exposed is dependent upon the firm's: A. tax rate. B. debt-equity ratio. C. return on assets. D. level of earnings before interest and taxes. E. operational level of risk.

B. debt-equity ratio

Which one of the following is the equity risk arising from the capital structure selected by a firm? A. Strategic risk B. Financial risk C. Liquidity risk D. Industry risk E. Business risk

B. financial risk

Which one of the following is an example of systematic risk? A. Major layoff by a regional manufacturer of power boats B. Increase in consumption created by a reduction in personal tax rates C. Surprise firing of a firm's chief financial officer D. Closure of a major retail chain of stores E. Product recall by one manufacturer

B. increase in consumption created by a reduction in personal tax rates

Which one of the following will decrease the aftertax cost of debt for a firm? A. Decrease in the firm's beta B. Increase in tax rates C. Increase in the risk-free rate of return D. Decrease in the market price of the debt E. Decrease in a bond's yield-to-maturity

B. increase in tax rates

A firm that uses its weighted average cost of capital as the required return for all of its investments will: A. maintain a constant value for its shareholders. B. increase the risk level of the firm over time. C. make the best possible accept and reject decisions related to those investments. D. find that its cost of capital declines over time. E. accept only the projects that add value to the firm's shareholders.

B. increase the risk level of the firm over time

A prepack: A. guarantees full payment to all creditors but lengthens the time span of the debt. B. is the joint filing of both a bankruptcy filing and a creditor-approved reorganization plan. C. protects the interests of both the current creditors and the existing shareholders. D. applies only if a firm files under Chapter 7 of the bankruptcy code. E. extends the time that a firm is protected by the bankruptcy process.

B. is the joint filing of both a bankruptcy filing and a creditor-approved reorganization plan

Which one of the following is a direct bankruptcy cost? A. Loss of customer goodwill resulting from a bankruptcy filing B. Legal and accounting fees related to a bankruptcy proceeding C. Management time spent on a bankruptcy proceeding D. Any financial distress cost E. Costs a firm spends trying to avoid bankruptcy

B. legal and accounting fees related to a bankruptcy proceeding

Which one of the following is the slope of the security market line? A. Risk-free rate B. Market risk premium C. Beta coefficient D. Risk premium on an individual asset E. Market rate of return

B. market risk premium

If the financial markets are semi-strong form efficient, then: A. only the most talented analysts can determine the true value of a security. B. only individuals with private information have a marketplace advantage. C. technical analysis provides the best tool to use to gain a marketplace advantage. D. no one individual has an advantage in the marketplace. E. every security offers the same rate of return.

B. only individuals with private information have a marketplace advantage

Which one of the following is a key provision of the Bankruptcy Abuse Prevention and Consumer Protection Act of 2005? A. Disallowance of bankruptcy prepacks B. Right granted to creditors to file their own reorganization plan once a firm is in bankruptcy for 18 months C. Disallowance of all management bonus payments while a firm is in bankruptcy D. Requirement that only creditors can file reorganization plans for a bankrupt firm E. Requirement for all Chapter 11 bankruptcies to be converted to Chapter 7 bankruptcies after 18 months

B. right granted to creditors to file their own reorganization plan once a firm is in bankruptcy for 18 months

Dan is a chemist for ABC, a major drug manufacturer. Dan cannot earn excess profits on ABC stock based on the knowledge he has related to his experiments if the financial markets are: A. weak form efficient. B. strong form efficient. C. semistrong form efficient. D. efficient at any level. E. aware that the trader is an insider.

B. strong form efficient

Which one of the following statements is correct? A. An increase in the market value of preferred stock will increase a firm's weighted average cost of capital. B. The cost of preferred stock is unaffected by the issuer's tax rate. C. Preferred stock is generally the cheapest source of capital for a firm. D. The cost of preferred stock remains constant from year to year. E. Preferred stock is valued using the capital asset pricing model.

B. the cost of preferred stock is unaffected by the issuer's tax rate

Consider a portfolio comprised of four risky securities. Assume the economy has three states with varying probabilities of occurrence. Which one of the following will guarantee that the portfolio variance will equal zero? A. The portfolio beta must be 1.0. B. The portfolio expected rate of return must be the same for each economic state. C. The portfolio risk premium must equal zero. D. The portfolio expected rate of return must equal the expected market rate of return. E. There must be equal probabilities that the state of the economy will be a boom or a bust.

B. the portfolio expected rate of return must be the same for each economic state

All else constant, an increase in a firm's cost of debt: A. could be caused by an increase in the firm's tax rate. B. will result in an increase in the firm's cost of capital. C. will lower the firm's weighted average cost of capital. D. will lower the firm's cost of equity. E. will increase the firm's capital structure weight of debt.

B. will result in an increase in the firm's cost of capital

All else constant, which of the following will increase the aftertax cost of debt for a firm? I. increase in the yield to maturity of the firm's outstanding debt II. decrease in the yield to maturity of the firm's outstanding debt III. increase in the firm's tax rate IV. decrease in the firm's tax rate A. I only B. I and III only C. I and IV only D. II and III only E. II and IV only

C. I and IV only

Which of the following terms can be used to describe unsystematic risk? I. asset-specific risk II. diversifiable risk III. market risk IV. unique risk A. I and IV only B. II and III only C. I, II, and IV only D. II, III, and IV only E. I, II, III, and IV

C. I, II, and IV only

The expected return on a security is currently based on a 22 percent chance of a 15 percent return given an economic boom and a 78 percent chance of a 12 percent return given a normal economy. Which of the following changes will decrease the expected return on this security? I. an increase in the probability of an economic boom II. a decrease in the rate of return given a normal economy III. an increase in the probability of a normal economy IV. an increase in the rate of return given an economic boom A. I and II only B. I and IV only C. II and III only D. I, III, and IV only E. I, II, III, and IV

C. II and III only

Which one of the following states that a firm's cost of equity capital is a positive linear function of the firm's capital structure? A. Static theory of capital structure B. M&M Proposition I C. M&M Proposition II D. Homemade leverage theory E. WACC

C. M&M Proposition II

Which one of the following supports the theory that the value of a firm increases as the firm's level of debt increases? A. M&M Proposition I, without taxes B. M&M Proposition II, without taxes C. M&M Proposition I, with taxes D. Static theory of capital structure E. No theory suggests this.

C. M&M proposition I, with taxes

Which one of the following statements is true regarding the period 1926-2008? A. The returns on small-company stocks were less volatile than the returns on large-company stocks. B. The risk-free rate of return remained constant over the time period. C. U.S. Treasury bills had a positive average real rate of return. D. Bonds had an average rate of return that exceeded the average return on stocks. E. The inflation rate was just as volatile as the return on long-term bonds.

C. U.S. treasury bills had a positive average real rate of return

The historical record for the period 1926-2008 shows that the annual nominal rate of return on: A. risk-free securities has averaged around 5 percent. B. the Consumer Price Index has been positive every year. C. U.S. Treasury bills have had a positive rate of return for every year in the period. D. U.S. Treasury bills is constant. E. large company stocks has averaged around 9 percent.

C. U.S. treasury bills have had a positive rate of return for every year in the period

According to the Efficient Market Hypothesis, professional investors will earn: A. excess profits over the long-term. B. excess profits, but only on short-term investments. C. a dollar return equal to the value paid for an investment. D.a return that cannot be accurately predicted because investments are subject to the random movements of the markets. E. a return that "beats the market".

C. a dollar return equal to the value paid for an investment

Which one of the following best defines legal bankruptcy? A. Negotiating new payment terms with a firm's creditors B. A temporary technical insolvency C. A legal proceeding for liquidating or reorganizing a business D. The internal process of revising the capital structure of a firm E. The failure of a firm to meet its financial obligations in a timely manner

C. a legal proceeding for liquidating or reorganizing a business

Which one of the following portfolios will have a beta of zero? A. A portfolio that is equally as risky as the overall market. B. A portfolio that consists of a single stock. C. A portfolio comprised solely of U. S. Treasury bills. D. A portfolio with a zero variance of returns. E. No portfolio can have a beta of zero.

C. a portfolio comprised solely of U.S. Treasury bill

Derek's is a brick-and-mortar toy store. The firm is considering expanding its operations to include Internet sales. Which one of the following would be the best firm to use in a pure play approach to analyzing this proposed expansion? A. Another brick-and-mortar store that also sells online B. A wholesale toy distributor C. A toy store that only sells online D. The oldest online retailer of any product E. Derek's own store

C. a toy store that only sells online

Semi-strong form market efficiency states that the value of a security is based on: A. all public and private information. B. historical information only. C. all publicly available information. D. all publicly available information plus any data that can be gathered from insider trading. E. random information with no clear distinction as to the source of that information.

C. all publicly available information

A firm has multiple divisions of similar nature, yet varying degrees of risk. Which one of the following would be the most appropriate, yet relatively easy, means of assigning discount rates to each of its proposed investments? A. Assign every project a rate equal to the firm's cost of equity B. Assign every firm a random rate that varies between the firm's cost of debt and its cost of equity C. Assign every project a rate equal to the firm's WACC plus or minus a subjective adjustment D. Determine the best pure play rate for each project E. Assign every project a rate equal to the market rate of return at the time of the proposal

C. assign every project a rate equal to the firm's WACC plus or minus a subjective adjustment

An efficient capital market is best defined as a market in which security prices reflect which one of the following? A. Current inflation B. A risk premium C. Available information D. The historical arithmetic rate of return E. The historical geometric rate of return

C. available information

The period 1926-2008 illustrates that U.S. Treasury bills: A. outperform inflation by approximately 1 percent every year. B. have a zero standard deviation. C. can either outperform or underperform inflation on an annual basis. D. produce a rate of return roughly equivalent to the rate of return on long-term government bonds. E. routinely have negative annual returns.

C. can either outperform or underperform inflation on an annual basis

The capital asset pricing model: A. assumes the market has a beta of zero. B. rewards investors based on total risk. C. considers the time value of money. D. applies to portfolios but not to individual securities. E. assumes the market risk premium is constant over time.

C. considers the time value of money

1. Katie owns 100 shares of ABC stock. Which one of the following terms is used to refer to the return that Katie and the other shareholders require on their investment in ABC? A. Weighted average cost of capital B. Pure play cost C. Cost of equity D. Subjective cost E. Cost of debt

C. cost of equity

M&M Proposition II, without taxes, states that the: A. capital structure of a firm is highly relevant. B. weighted average cost of capital decreases as the debt-equity ratio decreases. C. cost of equity increases as a firm increases its debt-equity ratio. D. return on equity is equal to the return on assets multiplied by the debt-equity ratio. E. return on equity remains constant as the debt-equity ratio increases.

C. cost of equity increases as a firm increases its debt-equity ratio

Which one of the following terms best refers to the practice of investing in a variety of diverse assets as a means of reducing risk? A. Systematic B. Unsystematic C. Diversification D. Security market line E. Capital asset pricing model

C. diversification

Which one of the following is the hypothesis that securities markets are efficient? A. Geometric market hypothesis B. Standard deviation hypothesis C. Efficient markets hypothesis D. Capital market hypothesis E. Financial markets hypothesis

C. efficient markets hypothesis

The use of borrowing by an individual to adjust his or her overall exposure to financial leverage is referred to as: A. M&M Proposition I. B. capital restructuring. C. homemade leverage. D. M&M Proposition II. E. financial risk management.

C. homemade leverage

Which one of the following will affect the capital structure weights used to compute a firm's weighted average cost of capital? A. Decrease in the book value of a firm's equity B. Decrease in a firm's tax rate C. Increase in the market value of the firm's common stock D. Increase in the market risk premium E. Increase in the firm's beta

C. increase in the market value of the firm's common stock

Which one of the following has the narrowest distribution of returns for the period 1926-2008? A. Long-term corporate bonds B. Long-term government bonds C. Intermediate-terms government bonds D. Large-company stocks E. Small-company stocks

C. intermediate-terms government bonds

Julie wants to create a $5,000 portfolio. She also wants to invest as much as possible in a high risk stock with the hope of earning a high rate of return. However, she wants her portfolio to have no more risk than the overall market. Which one of the following portfolios is most apt to meet all of her objectives? A. Invest the entire $5,000 in a stock with a beta of 1.0 B. Invest $2,500 in a stock with a beta of 1.98 and $2,500 in a stock with a beta of 1.0 C. Invest $2,500 in a risk-free asset and $2,500 in a stock with a beta of 2.0 D. Invest $2,500 in a stock with a beta of 1.0, $1,250 in a risk-free asset, and $1,250 in a stock with a beta of 2.0 E. Invest $2,000 in a stock with a beta of 3, $2,000 in a risk-free asset, and $1,000 in a stock with a beta of 1.0

C. invest $2,500 in a risk-free asset and $2,500 in a stock with a beta of 2.0

The static theory of capital structure assumes a firm: A. maintains a constant debt-equity ratio. B. has an all-equity structure. C. is fixed in terms of its assets. D. pays no taxes. E. is operating at the point where financial distress costs are eliminated.

C. is fixed in terms of its assets

Which one of the following is the primary determinant of an investment's cost of capital? A. Life of investment B. Initial cash outlay C. Level of risk D. Source of funds used for the investment E. Investment's net present value

C. level of risk

The addition of a risky security to a fully diversified portfolio: A. must decrease the portfolio's expected return. B. must increase the portfolio beta. C. may or may not affect the portfolio beta. D. will increase the unsystematic risk of the portfolio. E. will have no effect on the portfolio beta or its expected return.

C. may or may not affect the portfolio beta

Stock A comprises 28 percent of Susan's portfolio. Which one of the following terms applies to the 28 percent? A. Portfolio variance B. Portfolio standard deviation C. Portfolio weight D. Portfolio expected return E. Portfolio beta

C. portfolio weight

Peterboro recently defaulted on a bank loan. To avoid a bankruptcy proceeding, the bank agreed to a composition. This composition would do which one of the following? A. Forgive the loan payment in its entirety B. Extend the due date on the missed loan payment C. Reduce the amount of the loan payments so Peterboro can pay timely D. Transfer some of Peterboro's assets to the bank in lieu of the loan payment E. Transfer all the equity shares in Peterboro to the lending bank

C. reduce the amount of the loan payments so Peterboro can pay timely

Kelly's uses the firm's weighted average cost of capital (WACC) as the required return for some of its projects. For other projects, the firms uses a rate equal to WACC plus 1 percent, while another set of projects is assigned rates equal to WACC minus some amount. Which one of the following factors should be the key factor the firm uses to determine the amount of the adjustment it will make when assigning the project a discount rate? A. Firm beta B. Date for project commencement C. Risk level of project D. Division within the firm that will be assigned to manage the project E. Current debt-equity ratio

C. risk level of project

Which one of the following is most apt to cause a wise manager to increase a project's cost of capital? Assume the firm is levered. A. Management decides to issue new stock to finance the project. B. The initial cash outlay requirement is reduced. C. She learns the project is riskier than previously believed. D. The aftertax cost of debt just decreased. E. The project's life is shortened.

C. she learns the project is riskier than previously believed

Over the period of 1926-2008, which one of the following investment classes had the highest volatility of returns? A. Large-company stocks B. U.S. Treasury bills C. Small-company stocks D. Long-term corporate bonds E. Long-term government bonds

C. small-company stocks

5. Kate is the CFO of a major firm and has the job of assigning discount rates to each project that is under consideration. Kate's method of doing this is to assign an incrementally higher rate as the risk level of the project increases over that of the current firm. Likewise, she assigns lower rates as the risk level declines. Which one of the following approaches is Kate using to assign the discount rates? A. Pure play approach B. Divisional rating C. Subjective approach D. Straight WACC approach E. Equity rating

C. subjective approach

Which one of the following statements is correct related to the dividend growth model approach to computing the cost of equity? A. The rate of growth must exceed the required rate of return. B. The rate of return must be adjusted for taxes. C.The annual dividend used in the computation must be for year one if you are using today's stock price to compute the return. D. The cost of equity is equal to the return on the stock plus the risk-free rate. E. The cost of equity is equal to the return on the stock multiplied by the stock's beta.

C. the annual dividend used in the computation must be for year one if you are using today's stock price to compute the return

A firm has a return on equity of 12.4 percent according to the dividend growth model and a return of 18.7 percent according to the capital asset pricing model. The market rate of return is 13.5 percent. What rate should the firm use as the cost of equity when computing the firm's WACC? A. 12.4 percent because it is lower than 18.7 percent B. 18.7 percent because it is higher than 12.4 percent C. The arithmetic average of 12.4 percent and 18.7 percent D. The arithmetic average of 12.4 percent, 13.5 percent, and 18.7 percent E. 13.5 percent

C. the arithmetic average of 12.4 percent and 18.7 percent

Which one of the following is correct based on the static theory of capital structure? A. A firm receives the greatest benefit from debt financing when its tax rate is relatively low. B. A debt-equity ratio of 1 is considered to be the optimal capital structure. C. The costs of financial distress decrease the value of a firm. D. The more debt a firm assumes, the greater the incentive to acquire even more debt until such time as the firm is financed with 100 percent debt. E. At the optimal level of debt a firm also optimizes its tax shield on debt.

C. the costs of financial distress decrease the value of a firm

New Labs just announced that it has received a patent for a product that will eliminate all flu viruses. This news is totally unexpected and viewed as a major medical advancement. Which one of the following reactions to this announcement indicates the market for New Labs stock is efficient? A. The price of New Labs stock remains unchanged. B. The price of New Labs stock increases rapidly and then settles back to its pre-announcement level. C. The price of New Labs stock increases rapidly to a higher price and then remains at that price. D. All stocks quickly increase in value and then all but New Labs stock fall back to their original values. E. The value of all stocks suddenly increase and then level off at their higher values.

C. the price of new labs stock increases rapidly to a higher price and then remains at that price

Which one of the following best exemplifies unsystematic risk? A. Unexpected economic collapse B. Unexpected increase in interest rates C. Unexpected increase in the variable costs for a firm D. Sudden decrease in inflation E. Expected increase in tax rates

C. unexpected increase in the variable costs for a firm

The standard deviation measures the _____ of a security's returns over time. A. average value B. frequency C. volatility D. mean E. arithmetic average

C. volatility

Which one of the following is the pre-tax cost of debt? A. Average coupon rate on the firm's outstanding bonds B. Coupon rate on the firm's latest bond issue C. Weighted average yield-to-maturity on the firm's outstanding debt D. Average current yield on the firm's outstanding debt E. Annual interest divided by the market price per bond for the latest bond issue

C. weighted average yield-to-maturity on the firm's outstanding debt

When is a firm insolvent from an accounting perspective? A. When the firm is unable to meet its financial obligations in a timely manner B. When the firm's debt exceeds the value of the firm's equity C. When the firm has a negative net worth D. When the firm's revenues cease E. When the market value of the firm's equity equals zero

C. when the firm has a negative net worth

You are comparing two possible capital structures for a firm. The first option is an all-equity firm. The second option involves the use of $3.8 million of debt. The break-even point between these two financing options occurs when the earnings before interest and taxes (EBIT) are $428,000. Given this, you know that leverage is beneficial to the firm: A. whenever EBIT is less than $428,000. B. only when EBIT is $428,000. C. whenever EBIT exceeds $428,000. D. only if the debt is decreased by $428,000. E. only if the debt is increased by $428,000.

C. whenever EBIT exceeds $428,000

Marine Expeditors has three divisions. Division A is the core of the business and represents 80 percentof the firm's operations. Division B is involved only with contractual short-term projects and therefore has about 8 percent less risk than division A. Division C develops and markets new products and isabout 12 percent riskier than division A and about equal in size to division B. The manager of divisionA has suggested that the operations of his division be increased by 10 percent next year. The proposed project should probably be assigned a required return that is equal to _____ percent of the firm's weighted average cost of capital. A. 40 B. 60 C. 80 D. 100 E. 110

D. 100

What was the average annual risk premium on small-company stocks for the period 1926-2008? A. 5.3 percent B. 6.2 percent C. 8.5 percent D. 12.6 percent E. 15.3 percent

D. 12.6 percent

What is the probability associated with a return that lies in the upper tail when the mean plus two standard deviations is graphed? A. 0.05 percent B. 0.5 percent C. 1.0 percent D. 2.5 percent E. 5.0 percent

D. 2.5 percent

Which of the following features are advantages of the dividend growth model? I. easy to understand II. model simplicity III. constant dividend growth rate IV. model's applicability to all common stocks A. II only B. I and III only C. II and IV only D. I and II only E. I, II, and III only

D. I and II only

Based on the capital asset pricing model, investors are compensated based on which of the following? I. market risk premium II. portfolio standard deviation III. portfolio beta IV. risk-free rate A. I and III only B. II and IV only C. I, II, and III only D. I, III, and IV only E. I, II, III, and IV

D. I, III, and IV only

Which of the following are weaknesses of the dividend growth model? I. market risk premium fluctuations II. lack of dividends for some firms III. reliance on historical beta IV. sensitivity of model to dividend growth rate A. II only B. I and II only C. I and III only D. II and IV only E. I, II, III, and IV

D. II and IV only

The rate of return on which one of the following is used as the risk-free rate? A. Long-term government bonds B. Long-term corporate bonds C. Inflation, as measured by the Consumer Price Index D. U.S. Treasury bill E. Large-company stocks

D. U.S. treasury bill

For the period 1926-2008, which one of the following had the smallest risk premium? A. Large-company stocks B. Small-company stocks C. Long-term corporate bonds D. U.S. Treasury bills E. Long-term government bonds

D. U.S. treasury bills

Which one of the following conditions exists at the point where a firm maximizes its value? A. The tax benefit from an additional dollar of debt is zero. B. Financial distress costs are equal to zero. C. The debt-equity ratio is 1.0. D. WACC is minimized. E. The cost of equity is minimized.

D. WACC is minimized

A stock is expected to return 13 percent in an economic boom, 10 percent in a normal economy, and 3 percent in a recessionary economy. Which one of the following will lower the overall expected rate of return on this stock? A. An increase in the rate of return in a recessionary economy B. An increase in the probability of an economic boom C. A decrease in the probability of a recession occurring D. A decrease in the probability of an economic boom E. An increase in the rate of return for a normal economy

D. a decrease in the probability of an economic boom

Which one of the following statements related to the static theory of capital structure is correct? A. A firm begins to lose value as soon as the first dollar of debt is incurred. B. The actual value of a firm continually rises in direct proportion to the increased use of debt. C. The linear function of a firm's value has a constant positive slope. D. A firm's value is maximized when a firm operates at its optimal debt level. E. The value of a firm will automatically decrease whenever the debt-equity ratio is decreased.

D. a firm's value is maximized when a firm operates at its optimal debt level

Based on the period 1926-2008, what rate of return should you expect to earn over the long-term if you are unwilling to bear risk? A. Between 0 and 1 percent B. Between 1 and 2 percent C. Between 2 and 3 percent D. Between 3 and 4 percent E. Between 4 and 5 percent

D. between 3 and 4 percent

Which one of the following statements concerning financial leverage is correct? A. The benefits of leverage are unaffected by the amount of a firm's earnings. B. The use of leverage will always increase a firm's earnings per share. C. The shareholders of a firm are exposed to less risk anytime a firm uses financial leverage. D. Changes in the capital structure of a firm will generally change the firm's earnings per share. E. Financial leverage is beneficial to a firm only when the firm has negative earnings.

D. changes in the capital structure of a firm will generally change the firm's earnings per share

Which one of the following is the best example of systematic risk? A. Discovery of a major gas field B. Decrease in textile imports C. Increase in agricultural exports D. Decrease in gross domestic product E. Decrease in management bonuses for banking executives

D. decrease in gross domestic product

Which one of the following combinations will always result in an increased dividend yield? A. Increase in the stock price combined with a lower dividend amount B. Increase in the stock price combined with a higher dividend amount C. Decrease in the stock price combined with a lower dividend amount D. Decrease in the stock price combined with a higher dividend amount E. Increase in the stock price combined with a constant dividend amount

D. decrease in the stock price combined with a higher dividend amount

The security market line is defined as a positively sloped straight line that displays the relationship between which two of the following variables? A. Beta and standard deviation B. Systematic and unsystematic risk C. Nominal and real returns D. Expected return and beta E. Risk premium and beta

D. expected return and beta

Which one of the following statements concerning financial leverage is correct? A. Financial leverage increases profits and decreases losses. B. Financial leverage has no effect on a firm's return on equity. C. Financial leverage refers to the use of common stock. D. Financial leverage magnifies both profits and losses. E. Increasing financial leverage will always decrease the earnings per share.

D. financial leverage magnifies both profits and losses

You need to use the pure play approach to assign a cost of capital to a proposed investment. Which one of the following characteristics should you most concentrate on as you search for an appropriate pure play firm? A. Firm size B. Firm location C. Firm experience D. Firm operations E. Firm management

D. firm operations

Which one of the following best describes a portfolio? A. Risky security B. Security equally as risky as the overall market C. New issue of stock D. Group of assets held by an investor E. Investment in a risk-free security

D. group of assets held by an investor

The cost of capital for a project depends primarily on which one of the following? A. Source of funds used for the project B. Division within the firm that undertakes the project C. Project's modified internal rate of return D. How the project uses its funds E. Project's fixed costs

D. how the project uses its funds

Which one of the following statements is correct? A. The risk premium on a risk-free security is generally considered to be one percent. B. The expected rate of return on any security, given multiple states of the economy, must be positive. C. There is an inverse relationship between the level of risk and the risk premium given a risky security. D. If a risky security is correctly priced, its expected risk premium will be positive. E. If a risky security is priced correctly, it will have an expected return equal to the risk-free rate.

D. if a risky security is correctly priced, its expected risk premium will be positive

World United stock currently plots on the security market line and has a beta of 1.04. Which one of the following will increase that stock's rate of return without affecting the risk level of the stock, all else constant? A. An increase in the risk-free rate B. Decrease in the security's beta C. Overpricing of the stock in the market place D. Increase in the market risk-to-reward ratio E. Decrease in the market rate of return

D. increase in the market risk-to-reward ratio

A firm has a cost of equity of 13 percent, a cost of preferred of 11 percent, and an aftertax cost of debt of 6 percent. Given this, which one of the following will increase the firm's weighted average cost of capital? A. Increasing the firm's tax rate B. Issuing new bonds at par C. Redeeming shares of common stock D. Increasing the firm's beta E. Increasing the debt-equity ratio

D. increasing the firm's beta

Which one of the following is an example of a direct bankruptcy cost? A. Operating at a debt-equity ratio that is less than the optimal ratio B. Reducing the dividend payout ratio as a means of increasing a firm's equity C. Forgoing a positive net present value project to conserve current cash D. Incurring legal fees for the preparation of bankruptcy filings E. Losing a key customer due to concerns over a firm's financial viability

D. incurring legal fees for the preparation of bankruptcy filings

Paying interest reduces the taxes owed by a firm. Which one of the following terms applies to this relationship? A. Static theory of interest rates B. M&M Proposition I C. Financial risk D. Interest tax shield E. Homemade leverage

D. interest tax shield

The cost of preferred stock: A. increases when a firm's tax rate decreases. B. is constant over time. C. is unaffected by changes in the market price. D. is equal to the stock's dividend yield. E. increases as the price of the stock increases.

D. is equal to the stock's dividend yield

Kurt, who is a divisional manager, continually brags that his division's required return for its projects is one percent lower than the return required for any other division of the firm. Which one of the following most likely contributes the most to the lower rate requirement for Kurt's division? A. Kurt tends to overestimate the projected cash inflows on his projects. B. Kurt tends to underestimate the variable costs of his projects. C. Kurt has the most efficiently managed division. D. Kurt's division is less risky than the other divisions. E. Kurt's projects are generally financed with debt while the other divisions' projects are financed with equity.

D. kurt's division is less risky than the other divisions

The systematic risk principle states that the expected return on a risky asset depends only on which one of the following? A. Unique risk B. Diversifiable risk C. Asset-specific risk D. Market risk E. Unsystematic risk

D. market risk

Systematic risk is: A. totally eliminated when a portfolio is fully diversified. B. defined as the total risk associated with surprise events. C. risk that affects a limited number of securities. D. measured by beta. E. measured by standard deviation.

D. measured by beta

Which one of the following best describes an arithmetic average return? A. Total return divided by N - 1, where N equals the number of individual returns B. Average compound return earned per year over a multiyear period C. Total compound return divided by the number of individual returns D. Return earned in an average year over a multiyear period E. Positive square root of the average compound return

D. return earned in an average year over a multiyear period

T.L. C. Enterprises just revised its capital structure from a debt-equity ratio of 0.30 to a debt-equity ratio of 0.45. The firm's shareholders who prefer the old capital structure should: A. sell some shares and hold the sale proceeds in cash. B. sell all of their shares and loan out the entire sale proceeds. C. do nothing. D. sell some shares and loan out the sale proceeds. E. borrow funds and purchase more shares.

D. sell some shares and loan out the sale procedures

Which one of the following is the most apt to have the largest risk premium in the future based on the historical record for 1926-2008? A. U.S. Treasury bills B. Large-company stocks C. Long-term government debt D. Small-company stocks E. Long-term corporate debt

D. small-company stocks

Which one of the following is the best example of an announcement that is most apt to result in an unexpected return? A. A news bulletin that the anticipated layoffs by a firm will occur as expected on December 1 B. Announcement that the CFO of the firm is retiring June 1st as previously announced C. Announcement that a firm will continue its practice of paying a $3 a share annual dividend D. Statement by a firm that it has just discovered a manufacturing defect and is recalling its product E. The verification by senior management that the firm is being acquired as had been rumored

D. statement by a firm that it has just discovered a manufacturing defect and is recalling its product

Which one of the following could cause the total return on an investment to be a negative rate? A. Constant annual dividend amount B. Increase in the annual dividend amount C. Stock price that remains constant over the investment period D. Stock price that declines over the investment period E. Stock price that increases over the investment period

D. stock price that declines over the investment period

The risk premium for an individual security is based on which one of the following types of risk? A. Total B. Surprise C. Diversifiable D. Systematic E. Unsystematic

D. systematic

Which one of the following statements is the core principle of M&M Proposition I, without taxes? A. A firm's cost of equity is directly related to the firm's debt-equity ratio. B. A firm's WACC is directly related to the firm's debt-equity ratio. C. The interest tax shield increases the value of a firm. D. The capital structure of a firm is totally irrelevant. E. Levered firms have greater value than unlevered firms.

D. the capital structure of a firm is totally irrelevant

Which one of the following statements is correct? A. The risk-free rate of return has a risk premium of 1.0. B. The reward for bearing risk is called the standard deviation. C. Risks and expected return are inversely related. D. The higher the expected rate of return, the wider the distribution of returns. E. Risk premiums are inversely related to the standard deviation of returns.

D. the higher the expected rate of return, the wider the distribution of returns

7. Black Stone Furnaces wants to build a new facility. The cost of capital for this investment is primarily dependent upon which one of the following? A. Firm's overall source of funds B. Source of the funds used to build the facility C. Current tax rate D. The nature of the investment E. Firm's historical average rate of return

D. the nature of the investment

Which one of the following is the equity risk arising from the daily operations of a firm? A. Strategic risk B. Financial risk C. Liquidity risk D. Industry risk E. Business risk

E. business risk

Assume the securities markets are strong-form efficient. Given this assumption, you should expect which one of the following to occur? A. The risk premium on any security in that market will be zero. B. The price of any one security in that market will remain constant at its current level. C. Each security in the market will have an annual rate of return equal to the risk-free rate. D. The price of each security in that market will frequently fluctuate. E. The prices of each security will fall to zero because the net present value of the investments will be zero.

D. the price of each security in that market will frequently fluctuate

The average risk premium on long-term government bonds for the period 1926-2008 was equal to: A. zero. B. one percent. C. the rate of return on the bonds plus the corporate bond rate. D. the rate of return on the bonds minus the T-bill rate. E. the rate of return on the bonds minus the inflation rate.

D. the rate of return on the bonds minus the T-bill rate

Which one of the following statements is correct concerning capital structure weights? A. Target rates are less relevant to a project than are historical rates. B. The weights are unaffected when a bond issue matures. C. An increase in the debt-equity ratio will increase the weight of the common stock. D. The repurchase of preferred stock will increase the weight of debt. E. The issuance of additional shares of common stock will increase the weight of the preferred stock.

D. the repurchase of preferred stock will increase the weight of debt

Over the period of 1926-2008: A. the risk premium on large-company stocks was greater than the risk premium on small- company stocks. B. U.S. Treasury bills had a risk premium that was just slightly over 2 percent. C. the risk premium on long-term government bonds was zero percent. D. the risk premium on stocks exceeded the risk premium on bonds. E. U. S. Treasury bills had a negative risk premium.

D. the risk premium on stocks exceeded the risk premium on bonds

You are assigned the task of computing the expected return on a portfolio containing several individual stocks. Which one of the following statements is correct concerning this task? A. The expected rate of return on the portfolio must be positive. B. The arithmetic average of the betas for each security held in the portfolio must equal 1.0. C. The portfolio beta must be 1.0. D. The summation of the return deviation from the portfolio expected return for each economic state must equal zero. E. The standard deviation of the portfolio must equal 1.0.

D. the summation of the return deviation from the portfolio expected return for each economic state must equal zero

Which one of the following statements matches M&M Proposition I? A. The cost of equity capital has a positive linear relationship with a firm's capital structure. B. The dividends paid by a firm determine the firm's value. C. The cost of equity capital varies in response to changes in a firm's capital structure. D. The value of a firm is independent of the firm's capital structure. E. The value of a firm is dependent on the firm's capital structure.

D. the value of a firm is independent of the firm's capital structure

The expected rate of return on Delaware Shores, Inc. stock is based on three possible states of the economy. These states are boom, normal, and recession which have probabilities of occurrence of 20 percent, 75 percent, and 5 percent, respectively. Which one of the following statements is correct concerning the variance of the returns on this stock? A. The variance must decrease if the probability of occurrence for a boom increases. B. The variance will remain constant as long as the sum of the economic probabilities is 100 percent. C.The variance can be positive, zero, or negative, depending on the expected rate of return assigned to each economic state. D. The variance must be positive provided that each state of the economy produces a different expected rate of return. E. The variance is independent of the economic probabilities of occurrence.

D. the variance must be positive provided that each state of the economy produces a different expected rate of return

Standard deviation measures _____ risk while beta measures _____ risk. A. systematic; unsystematic B. unsystematic; systematic C. total; unsystematic D. total; systematic E. asset-specific; market

D. total; systematic

Portfolio diversification eliminates which one of the following? A. Total investment risk B. Portfolio risk premium C. Market risk D. Unsystematic risk E. Reward for bearing risk

D. unsystematic risk

Which one of the following represents the rate of return a firm must earn on its assets if it is to maintain the current value of its securities? A. Cost of equity B. Internal rate of return C. Aftertax cost of debt D. Weighted average cost of capital E. Debt-equity ratio

D. weighted average cost of capital

Which one of the following is the computation of the risk premium for an individual security? E(r) is the expected return on the security, rf is the risk-free rate, β is the security's beta, and E(r)M is the expected rate of return on the market. A. E(r)M - rf B. E(r) - E(r)M C. E(r) - (E(r)M + rf) D. β[E(r)M - rf] E. β[E(r) - rf]

D. β[E(r)M - rf]

Percentage returns: I. are easy to understand. II. relay information about a security more easily than dollar returns do. III. are not affected by the amount of the investment. IV. can be easily separated into dividend yield and capital gain yield. A. II and III only B. I and III only C. I, II, and III only D. I, II, and IV only E. I, II, III, and IV

E. I, II, III, and IV

Which one of the following is the minimum required rate of return on a new investment that makes that investment attractive? A. Risk-free rate B. Market risk premium C. Expected return minus the risk-free rate D. Market rate of return E. Cost of capital

E. cost of capital

The security market line is a linear function which is graphed by plotting data points based on the relationship between which two of the following variables? A. Risk-free rate and beta B. Market rate of return and beta C. Market rate of return and the risk-free rate D. Risk-free rate and the market rate of return E. Expected return and beta

E. expected return and beta

Which one of the following represents the present value of the interest tax shield? A. D×(1-Tc) B. D/(1 - Tc) C. D/Tc D. D - D(Tc) E. TC × D

E. TC × D

Based on the capital asset pricing model, which one of the following must increase the expected return on an individual security, all else constant? A. An increase in the risk level of that security as measured by the standard deviation B. An increase in the risk-free rate given a security beta of 1.42 C. A decrease in the market rate of return given a security beta of 1.13 D. A decrease in the market rate of return given a security beta of .78 E. A decrease in the risk-free rate given a security beta of 1.06

E. a decrease in the risk-free rate given a security beta of 1.06

Which one of the following statements is correct? A. All Chapter 7 bankruptcy filings must include a "workout" agreement. B. Firms must remain in bankruptcy for at least 18 months. C. Key employee retention plans (KERPS) are no longer legal. D. Labor contracts cannot be modified through the bankruptcy process. E. A firm can file for Chapter 11 bankruptcy even if the firm is solvent.

E. a firm can file for chapter 11 bankruptcy even if the firm is solvent

Which one of the following statements is accurate for a levered firm? A. WACC should be used as the required return for all proposed investments. B. A firm's WACC will decrease whenever the firm's tax rate decreases. C. An increase in the market risk premium will decrease a firm's WACC. D. The subjective approach totally ignores a firm's own WACC. E. A reduction in the risk level of a firm will tend to decrease the firm's WACC.

E. a reduction in the risk level of a firm will tend to decrease the firm's WACC

Which one of the following statements is correct? A. A portfolio that contains at least 30 diverse individual securities will have a beta of 1.0. B. Any portfolio that is correctly valued will have a beta of 1.0. C. A portfolio that has a beta of 1.12 will lie to the left of the market portfolio on a security market line graph. D. A risk-free security plots at the origin on a security market line graph. E. An underpriced security will plot above the security market line.

E. an underpriced security will plot above the security market line

Which one of the following will generally receive the highest priority in a bankruptcy liquidation, assuming the absolute priority rule is followed? A. Claims by unsecured creditors B. Employee wages C. Government tax claims D. Contributions to employee retirement plans E. Bankruptcy administrative expenses

E. bankruptcy administrative expenses

When using the pure play approach for a proposed investment, a firm is primarily seeking a rate of return which: A. is based on the actual source of funds that will be used to fund the project. B. creates a positive net present value for the project. C. reflects the size and life of the project. D. most closely correlates with the proposed investment's internal rate of return. E. best matches the risk level of the proposed investment.

E. best matches the risk level of the proposed investment

In an efficient market, the cost of equity for a risky firm does which one of the following according to the security market line? A. Produces a return that will be less than the market rate but higher than the risk-free rate B. Equals the market rate of return for all stocks C. Has a maximum cost equal to the market rate of return D. Decreases as the beta of the firm's stock increases E. Increases in direct relation to the stock's systematic risk

E. increases in direct relation to the stock's systemic risk

Which one of the following terms refers to the termination of a firm as a going concern? A. Insolvency B. Reorganization C. Chapter 11 bankruptcy D. Prepack E. Liquidation

E. liquidation

When, if ever, will the geometric average return exceed the arithmetic average return for a given set of returns? A. When the set of returns includes only risk-free rates. B. When the set of returns has a wide frequency distribution. C. When the set of returns has a very narrow frequency distribution. D. When all of the rates of return in the set of returns are equal to each other. E. Never

E. never

Which one of the following is defined as a bell-shaped frequency distribution that is defined by its average and its standard deviation? A. Arithmetic average return B. Variance C. Standard deviation D. Probability curve E. Normal distribution

E. normal distribution

Assume you own a portfolio of diverse securities which are each correctly priced. Given this, the reward- to-risk ratio: A. for the portfolio must equal 1.0. B. for the portfolio must be less than the market risk premium. C. for each security must equal zero. D. of each security is equal to the risk-free rate. E. of each security must equal the slope of the security market line.

E. of each security must equal the slope of the security market line

4. Farmer's Supply, Inc. is considering opening a clothing store, which would be a new line of business for the firm. Management has decided to use the cost of capital of a similar clothing store as the discount rate that should be used to evaluate this proposed expansion. Which one of the following terms is used to describe the approach Farmer's Supply is taking to establish an appropriate discount rate for the project? A. Equity approach B. Aftertax approach C. Subjective approach D. Market play E. Pure play approach

E. pure play approach

Diversifying a portfolio across various sectors and industries might do more than one of the following. However, this diversification must do which one of the following? A. Increase the expected risk premium B. Reduce the beta of the portfolio to zero C. Increase the security's risk premium D. Reduce the portfolio's systematic risk level E. Reduce the portfolio's unique risks

E. reduce the portfolio's unique risks

Greenwood Motels has filed a petition for bankruptcy but hopes to continue its operations both during and after the bankruptcy process. Which one of the following terms best applies to this situation? A. Chapter 7 bankruptcy B. Liquidation C. Technical insolvency D. Accounting insolvency E. Reorganization

E. reorganization

Investors require a 4 percent return on risk-free investments. On a particular risky investment, investors require an excess return of 7 percent in addition to the risk-free rate of 4 percent. What is this excess return called? A. Inflation premium B. Required return C. Real return D. Average return E. Risk premium

E. risk premium

Which one of the following is the vertical intercept of the security market line? A. Market rate of return B. Individual security rate of return C. Market risk premium D. Individual security beta multiplied by the market risk premium E. Risk-free rate

E. risk-free rate

Which one of the following is the theory that a firm should borrow up to the point where the additional tax benefit from an extra dollar of debt equals the additional costs associated with financial distress from that additional debt? A. M&M Proposition I, with taxes B. M&M Proposition II, with taxes C. M&M Proposition I, without taxes D. Homemade leverage proposition E. Static theory of capital structure

E. static theory of capital structure

If the financial markets are efficient then: A. stock prices should remain constant. B.stock prices should increase or decrease slowly as new events are analyzed and the information is absorbed by the markets. C. an increase in the value of one security should be offset by a decrease in the value of another security. D. stock prices will only change when an event actually occurs, not at the time the event is anticipated. E. stock prices should only respond to unexpected news and events.

E. stock prices should only respond to unexpected news and events

The historical returns on large-company stocks, as reported by Ibbotson and Sinquefield and reported in your textbook, are based on the: A. largest 20 percent of the stocks traded on the NYSE. B. stock returns for the largest 10 percent of the publicly traded firms in the U.S. C. returns of the 100 largest firms in the U.S. D. returns of all of the stocks listed on the NYSE. E. stocks of the 500 companies included in the S&P 500 index.

E. stocks of the 500 companies included in the S&P 500 index

The computation of which one of the following requires assigning every proposed investment to a particular risk class? A. Pure play cost of capital B. Cost of equity C. Aftertax cost of debt D. WACC E. Subjective cost of capital

E. subjective cost of capital

One year ago, you purchased 100 shares of a stock .This morning you sold those shares and realized a total return of 8.2 percent. Given this information, you know for sure the: A. stock price increased by 8.2 percent over the last year. B. stock increased in value over the past year. C. stock paid a dividend. D. dividend yield is greater than zero. E. sum of the dividend yield and the capital gains yield is 8.2 percent.

E. sum of the dividend yield and the capital gains yield is 8.2 percent

The beta of a risky portfolio cannot be less than _____ nor greater than _____. A. 0; 1 B. 1; the market beta C. the lowest individual beta in the portfolio; market beta D. the market beta; the highest individual beta in the portfolio E. the lowest individual beta in the portfolio; the highest individual beta in the portfolio

E. the lowest individual beta in the portfolio; the highest individual beta in the portfolio

Which one of the following is an implication of M&M Proposition II, without taxes? A. A firm's optimal capital structure is 100 percent debt. B. WACC is unaffected by the capital structure of a firm. C. WACC decreases as the debt-equity ratio increases. D. A firm's capital structure is irrelevant. E. The risk of equity depends on both the degree of financial leverage and the riskiness of the firm's operations.

E. the risk of equity depends on both the degree of financial leverage and the riskiness of the firm's operations

Assume you are comparing two firms that are identical in every aspect, except one is levered and one is unlevered. Which one of the following statements is correct regarding these two firms? A. The levered firm has higher EPS than the unlevered firm at the break-even point. B. The levered firm will have higher EPS than the unlevered firm at all levels of EBIT. C. The unlevered firm will have higher EPS than the levered firm at relatively high levels of EBIT. D. The EPS for the unlevered firm will always exceed those of the levered firm. E. The unlevered firm will have higher EPS at relatively low levels of EBIT.

E. the unlettered firm will have higher EPS at relatively low levels of EBIT

A portfolio is comprised of 35 securities with varying betas. The lowest beta for an individual security is 0.74 and the highest of the security betas of 1.51. Given this information, you know that the portfolio beta: A. must be 1.0 because of the large number of securities in the portfolio. B. is the geometric average of the individual security betas. C. must be less than the market beta. D. will be between 0 and 1.0. E. will be greater than or equal to 0.74 but less than or equal to 1.51.

E. will be greater than or equal to 0.74 but less than or equal to 1.51

Which one of the following represents the amount of compensation an investor should expect to receive for accepting the unsystematic risk associated with an individual security? A. Security beta multiplied by the market rate of return B. Market risk premium C. Security beta multiplied by the market risk premium D. Risk-free rate of return E. Zero

E. zero


Conjuntos de estudio relacionados

final exam immunology review (multiple choice)

View Set

Cuál es el rol de la monarquía en Reino Unido

View Set